If exactly two judges voted against Datalog, then which one of the following must be true?

Caden on June 14 at 01:04AM

Why is A correct?

Hi, I am wondering why both Ms have to go in the "For" category. If "Against" has exactly two slots and one of them is filled by C, couldn't one of the Ms fill the other slot in "Against"? Or am I doing the setup for this question incorrectly? Thank you so much, Caden

Replies
Create a free account to read and take part in forum discussions.

Already have an account? log in

Emil-Kunkin on June 14 at 01:38AM

Hi, I think your setup is correct. We would need to look at the implications of what would happen if we had an M in the second no slot.

This would look like
Y: M L L L C
N: M C

This would violate the rule that if three liberals agree, no conservative can agree with them. So, we must have either one of the three liberals or the second conservative in the no group if there are exactly two nos.

Caden on June 14 at 02:48PM

Perfect, thank you so much!